a battleship simultaneously fires two shells at enemy ships. if the shells follow the parabolic trajectories shown, which ship gets hit first?

Answers

Answer 1

A battleship simultaneously fires two shells in parabolic projectile motion and no information about initial speeds at enemy ships. The ship B got hit first. So, the correct choice for answer is option (c).

Here is we have a battleship Which fires two shells simultaneously at the enemy ship along the two paths. The initial speed of projection may be same or different. See the above figure carefully, the angle of projection for ship A is more than ship B. Time of flight for ship A is

[tex]T_A = \frac{ 2u_{A} sinθ_{A}}{g }[/tex]

For ship B, [tex]T_B = \frac{2u_B sinθ_{B}}{g }[/tex]

We have no idea about the initial speed of projection, so we cannot consider it for comparison. As we know from above,

[tex]θ_{A} > θ_{B}[/tex]

=> [tex]sinθ_{A} > sinθ_{B}[/tex]

So, [tex]T_{A} > T_{B}[/tex]

That is time of flight for ship A is greater than for the ship B. Therefore, ship B gets hit first.

For more information about projectile motion, visit :

https://brainly.com/question/28687429

#SPJ4

Complete question:

A battleship simultaneously fires two shells at enemy ships. if the shells follow the parabolic trajectories shown, which ship gets hit first?

a) A

b) both simultaneously

c) B

d) None

A Battleship Simultaneously Fires Two Shells At Enemy Ships. If The Shells Follow The Parabolic Trajectories

Related Questions

Suppose a NASCAR race car rounds one end of the Martinsville Speedway. This end of the track is a turn with a radius of approximately 57.0 m . If the track is completely flat and the race car is traveling at a constant 27.5 m/s (about 62 mph ) around the turn, what is the race car's centripetal (radial) acceleration? What is the Coefficient of friction?

Answers

Answer:

Explanation:

The centripetal acceleration of the race car is given by the formula:

a = v^2 / r

where v is the speed of the race car and r is the radius of the turn.

Substituting the given values, we get:

a = (27.5 m/s)^2 / 57.0 m = 13.3 m/s^2

So the centripetal acceleration of the race car is 13.3 m/s^2.

To find the coefficient of friction, we need to use the formula:

f = μN

where f is the force of friction, μ is the coefficient of friction, and N is the normal force.

The normal force is equal to the weight of the car, which we can calculate as:

N = mg

where m is the mass of the car and g is the acceleration due to gravity (9.81 m/s^2).

Assuming the mass of the car is 1500 kg, we get:

N = 1500 kg × 9.81 m/s^2 = 14,715 N

The force of friction is equal to the centripetal force required to keep the car moving in a circle:

f = ma = (1500 kg)(13.3 m/s^2) = 19,950 N

Substituting the values of N and f into the formula for friction, we get:

19,950 N = μ(14,715 N)

Solving for μ, we get:

μ = 1.35

So the coefficient of friction is 1.35.

In this circuit, what is the potential difference across C4?
Use the following values in your calculation:
V = 12.0 V
C1 = 3.0 ?F
C2 = 2.0 ?F
C3 = 2.0?F
C4 = 1.0 ?F
C5 = 4.0 ?F
V4 =

Answers

The potential difference across C4 can be found using the equation V = V4 - V3. Using the given values, V = 12.0V, C1 = 3.0 ?F, C2 = 2.0 ?F, C3 = 2.0 ?F, C4 = 1.0 ?F, and C5 = 4.0 ?F, we can solve for V4.



V4 = 12.0V + (3.0 ?F + 2.0 ?F + 2.0 ?F + 1.0 ?F) / (1.0 ?F + 4.0 ?F)
V4 = 12.0V + (8.0 ?F / 5.0 ?F)
V4 = 12.0V + 1.6V
V4 = 13.6V
Therefore, the potential difference across C4 is 13.6V - 12.0V = 1.6V.
The potential difference across C4 can be determined using the formula Q = CV. Where Q represents the charge stored in the capacitor, C represents capacitance, and V represents the potential difference across the capacitorTo determine the potential difference across C4, we can use the formula Q = CV. To determine Q, we need to determine the equivalent capacitance of the circuit.

The equivalent capacitance of capacitors in parallel is equal to the sum of their capacitance. The equivalent capacitance of capacitors in series is equal to the reciprocal of the sum of their reciprocals.C1, C2, and C3 are in series, and their equivalent capacitance is given by:C_eq1=1/((1/C1)+(1/C2)+(1/C3))=1/(1/3+1/2+1/2)=3/7 μF{C_eq1=1/((1/C1)+(1/C2)+(1/C3))=1/(1/3+1/2+1/2)=3/7μF}C_eq2 is the equivalent capacitance of C4 and C5 in parallel.C_eq2=C4+C5=1+4=5μF {C_eq2=C4+C5=1+4=5μF}

Now we can determine the equivalent capacitance of the entire circuit.C_eq=C_eq1+C_eq2=3/7+5=38/7μF{C_eq=C_eq1+C_eq2=3/7+5=38/7μF}Now, we can determine the charge stored in the circuit.Q=C_eqV=38/7*12= 65.14μC{Q=C_eqV=38/7*12=65.14μC}To determine the potential difference across C4, we can use the formula Q = CV.V=C4Q/C4= 65.14/1 = 65.14V{V=C4Q/C4=65.14/1=65.14V}Therefore, the potential difference across C4 is 65.14 V.

For more such questions on potential difference

https://brainly.com/question/3535763

#SPJ11

Commercially available large wind turbines blade span diameters larger than 100 m and over 3 MW of electric power at peak design have generate conditions. Consider a wind turbine with a 75-m blade span subjected to 25-km/h steady winds. If the combined turbine–generator effi- ciency of the wind turbine is 32 percent, determine (a) the power generated by the turbine and (b) the horizontal force exerted by the wind on the supporting mast of the turbine. Take the density of air to be 1.25 kg/m3, and disregard frictional effects on the mast.

Answers

The horizontal force that was exerted by the wind on the mast based on the power is 67.3KN.

What is the force?

Blade Stan, d = 75m

Radius of Blade, r = 75m

wind velocity, V = 30 km/h V = 8.333 m/s

Turbine Generator efficiency or Power Co-efficient ((p) = 32% 0.32.

Flow rate across the turbine (in) = 125X8.333X X (75) 2 m

= 46017.583 kg/s

Air Exit velocity, Ve = V×√1 - Nterbine

Ve = 8.333 x √1 1- 0.32

Ve = 6.872 mls

Horizental force in x-direction (F); -

Fx = m (ve-v)

Fx = 46017-583X(6-872-8.333) = 67265.381 N

The Horizental force Extered on the Supporting mast F = -F F= 67.2654 KN

Learn more about power on:

https://brainly.com/question/2248465

#SPJ1

Determine the horizontal force that was exerted by the wind on the mast base

What are density and volume?

Simple explanation please​

Answers

Answer:

Explanation:

Density is a measure of how much mass is contained in a given volume. It is the amount of matter (mass) in a given space (volume). Density is usually expressed in units of mass per unit of volume, such as kilograms per cubic meter (kg/m³) or grams per milliliter (g/mL).

Volume is the amount of space occupied by an object or substance. It is the measurement of the three-dimensional space occupied by an object, substance, or material. Volume can be measured in different units, such as liters (L), cubic meters (m³), or cubic feet (ft³), depending on the scale of the object being measured.

Answer ....Volume refers to the measurement of the amount of three-dimensional space occupied by an object. Unlike mass, volume changes according to the external conditions. Density refers to the mass contained in a substance for a given volume. It explains the relationship between mass and volume

Pete needs to be at work for 9.00am. He leaves his house at 7.30am and drives to the gym which is 12.5 miles away. Pete spends 45 minutes in the gym then drives the reaming 9 miles to work.

Answers

To determine the time Pete arrives at work, we can start by calculating the total time he spends on his commute and gym routine:

What time will Pete get to work?

Time spent driving to the gym = 12.5 miles ÷ average speed

We don't know Pete's average speed, so we cannot calculate this.

Time spent in the gym = 45 minutes

Time spent driving from the gym to work = 9 miles ÷ average speed

Again, we don't know Pete's average speed, so we cannot calculate this.

Total time spent on commute and gym routine = time spent driving to gym + time spent in gym + time spent driving from gym to work

= Unknown + 45 minutes + Unknown

Next, we can convert the total time to hours and minutes:

Total time = (Unknown + 45 minutes + Unknown) ÷ 60

= (Unknown + Unknown) ÷ 60 + 45/60

= (2Unknown) ÷ 60 + 0.75

= (Unknown) ÷ 30 + 0.75

We know that Pete needs to arrive at work by 9.00am, so we can set up an equation:

Arrival time = 7.30am + Total time

9.00am = 7.30am + (Unknown/30) + 0.75

Solving for Unknown:

1.5 hours = Unknown/30

Unknown = 45 minutes

Therefore, Pete will arrive at work at 8.15am.

Learn more about time from

https://brainly.com/question/18252403

#SPJ1

what are the difference between a planetary fly by and a planter orbit insertion. list 6 thing for each, find the answer for NASA.gov​

Answers

Answer:

Explanation:

Planetary Flyby:

The spacecraft does not go into orbit around the planet; instead, it uses the planet's gravity to change its speed and direction.

The spacecraft's closest approach to the planet is usually brief, ranging from a few minutes to a few hours.

The spacecraft is able to capture images and data during the brief encounter with the planet.

The spacecraft's trajectory can be adjusted to perform multiple flybys of different planets or moons.

The spacecraft does not require a large amount of fuel to perform a flyby, making it a cost-effective option for exploration.

Flybys are useful for studying a planet's atmosphere, magnetic field, and gravitational field.

Planetary Orbit Insertion:

The spacecraft goes into orbit around the planet, allowing for long-term study and data collection.

The spacecraft's orbit can be adjusted to achieve different scientific objectives, such as mapping the planet's surface or studying its atmosphere.

The spacecraft must have enough fuel to slow down and enter orbit, making it a more expensive option than a flyby.

The spacecraft's orbit can be stable or elliptical, depending on the scientific objectives and mission requirements.

The spacecraft may require several trajectory adjustments to achieve the desired orbit.

Orbit insertion allows for more detailed and comprehensive study of a planet's geology, climate, and magnetic field.

At one instant an object in free fall is moving downward at 50 meters per second. One second later its speed is about
A) 25 m/s. B) 50 m/s. C) 55 m/s. D) 60 m/s. E) 100 m/s.

Answers

The correct answer is C) 55 m/s. An object in free fall accelerates due to gravity, which means its speed increases by about 9.8 m/s2 every second. So in one second, its speed increased from 50 m/s to 50 + 9.8 = 59.8 m/s. Since it is impossible for the object to have a speed of 59.8 m/s, the closest answer is C) 55 m/s.


Given,An object in free fall is moving downward at 50 meters per second.At one-second later its speed is about.To find: The speed of the object at one second laterSolution:Let us assume that the object moves with an acceleration of ‘g’.Given, Initial velocity, u = 50 m/s

Time taken, t = 1sWe know that the velocity of an object in freefall is given by:v = u + gtFrom the above equation, we can calculate the final velocity of the object after one secondv = u + gtv = 50 + 9.8 × 1v = 50 + 9.8v = 59.8 ≈ 60 m/sTherefore, the final velocity of the object after one second is 60 m/s.Hence, the correct option is (D) 60 m/s.

For more such questions on accelerates

https://brainly.com/question/30649277

#SPJ11

Estimat the number and wattage of lamps. which would be required to illuminate a workshop space 60x1.5 meteres by means of lamps mounted 5 metres above the working Plane The average illumination required is about 100 wt. coefficient of utilisation = 0.4 luminous efficiency 16 lumens per watt. Assume a space-height ratio of unity and a cundle Power depreciation of 20%​

Answers

The number and wattage of lamps required to illuminate the workshop would be approximately 8 lamps and 70 watts respectively.

Wattage calculation

To estimate the number and wattage of lamps required to illuminate a workshop space of 60x1.5 meters, we can follow these steps:

Calculate the area of the workshop:

Area = length x widthArea = 60m x 1.5mArea = 90 square meters

Determine the total lumens required:

Lumens = area x average illuminationLumens = 90 sq m x 100 luxLumens = 9000 lumens

Adjust for the coefficient of utilization and luminous efficiency:

Effective lumens = lumens / (coefficient of utilization x luminous efficiency)Effective lumens = 9000 / (0.4 x 16)Effective lumens = 1406.25 lumens

Adjust for space-height ratio and candle power depreciation:

Effective lumens per lamp = effective lumens x space-height ratio x (1 - depreciation)Effective lumens per lamp = 1406.25 x 1 x (1 - 0.2)Effective lumens per lamp = 1125 lumens

Determine the number of lamps required:

Number of lamps = total lumens required / effective lumens per lampNumber of lamps = 9000 / 1125Number of lamps = 8 lamps (rounded up)

Determine the wattage of each lamp:

Wattage per lamp = effective lumens per lamp / luminous efficiencyWattage per lamp = 1125 / 16Wattage per lamp = 70.3 watts (rounded up)

Therefore, approximately 8 lamps with a wattage of 70 watts each would be required to illuminate the workshop space.

More on wattage can be found here: https://brainly.com/question/14667843

#SPJ1

A beam consisting of five types of ions labeled A, B, C, D, and E enters a region that contains a uniform magnetic field as shown in the figure below. The field is perpendicular to the plane of the paper, but its precise direction is not given. All ions in the beam travel with the same speed. The table below gives the masses and charges of the ions. Note: 1 mass unit = 1.67 x 10â€"27 kg and e = 1.6 x 10â€"19 C
Which ion falls at position 2?

Answers

At position 2, ion B falls. It is less deflected because it has a lesser mass than ions C, D, and E and the same charge as ion A.

A force perpendicular to the charged particle's velocity and the magnetic field's direction is applied when it reaches the magnetic field. The right-hand rule asserts that the palm will face the direction of the force if the thumb of the right hand points in the direction of the particle's velocity and the fingers point in the direction of the magnetic field. The particle's charge, velocity, and magnetic field intensity all affect how much force is generated.

Since all ions are moving at the same speed in this scenario, the force exerted on each ion is proportional to its charge to mass ratio. Ion B has the smallest mass of all the ions, so the least force and is least deflected of the ions, falling at position 2.

learn more about mass here:

https://brainly.com/question/19694949

#SPJ4

Select the correct answer. In a given chemical reaction, the energy of the products is greater than the energy of the reactants. Which statement is true for this reaction? A. Energy is absorbed in the reaction. B. Energy is released in the reaction. C. No energy is transferred in the reaction. D. Energy is created in the reaction. E. Energy is lost in the reaction. Reset Next

Answers

A - energy is absorbed during the reaction

3. Large amplitude vibrations produced when the of receiver of the applied forced vibration matches the

Answers

An object's amplitude dramatically increases when the frequency of the applied forced vibrations matches the object's natural frequency. Resonance describes this behavior.

Theory A wave's amplitude directly relates to the quantity of energy it can carry. A wave with a high amplitude carries a lot of energy, whereas one with a low amplitude carries only a little. A wave's strength is determined by the typical energy that moves through a given area in a certain amount of time and in a particular direction.The sound wave's amplitude grows in proportion to its strength. We perceive louder noises to be of higher intensity. Comparative sound intensities are frequently expressed using decibels (dB)

For more information on amplitude of vibration kindly visit to

https://brainly.com/question/1380029

#SPJ1

Two pieces of clay, one white and one gray, are thrown through the air. The
m
white clay has a momentum of 25 kg, and the gray clay has a
S
momentum of -30 kg immediately before they collide.
What is the magnitude and direction of their final momentum immediately
after the collision?
Your answer should have one significant figure.
h
kg.
m
-
m
S
S

Answers

we can't give a specific direction for the final momentum.

What is momentum?

Momentum is a physical quantity that describes the motion of an object. It is defined as the product of an object's mass and its velocity. Mathematically, momentum is expressed as:

Momentum (p) = mass (m) x velocity (v)

p = m x v

To solve this problem, we need to apply the law of conservation of momentum, which states that the total momentum of a system remains constant if no external forces act on it.

The initial total momentum of the system is:

p_initial = p_white + p_gray = 25 kg m/s - 30 kg m/s = -5 kg m/s

Since there are no external forces acting on the system, the total momentum of the system after the collision must also be -5 kg m/s. Therefore, the final momentum of the system is:

p_final = -5 kg m/s

The direction of the final momentum can be found by looking at the directions of the initial momenta. Since the white clay has positive momentum and the gray clay has negative momentum, we can say that the white clay is moving to the right and the gray clay is moving to the left before the collision.

During the collision, the two clays will exert forces on each other, causing them to change direction and possibly even break apart. Without more information about the collision, we can't say for sure what the direction of the final momentum will be. It could be to the left or to the right, or some combination of the two. Therefore, we can't give a specific direction for the final momentum.

To know more about Magnitude visit:-

https://brainly.com/question/24468862

#SPJ1

Two long parallel wires placed side by side on a horizontal table carry the same currents in opposite directions. The wire on your right carries current toward you, and the wire on your left carries current away from you. Determine the direction of the magnetic field at the point exactly midway between the two wires from your point of view. Explain your answer with the aid of labelled diagram. [5 marked​

Answers

To find:-

Magnetic field at the centre between the wires.

Answer:-

We are here given that two long current carrying wires are having same current. We need to find out the magnetic field at the centre between the wires .

We know that for a point between two ends of a wire , magnetic field is given by,

[tex]\implies B =\dfrac{\mu_0}{4\pi}\dfrac{2i}{d}\\[/tex]

where ,

B is magnetic field.i is the current.d is the distance .

Now since magnetic field is a vector quantity we need to find out the direction of the field . We can do so by using Right Hand thumb rule .

Right hand thumb rule :-

Hold the wire , in your hand with thumbs towards the direction of the current, then the curling of the fingers would give you the direction of the magnetic field.

For wire AB :-

The direction comes to be down the page .

For wire CD :-

The direction comes to be down the page .

Calculating net magnetic field:-

The net magnetic field will be the sum of both the fields .

[tex]\implies B_{net}=\dfrac{\mu_0}{4\pi}\dfrac{2i}{d}+\dfrac{\mu_0}{4\pi}\dfrac{2i}{d} \\[/tex]

[tex]\implies B_{net}=\dfrac{\mu_0}{4\pi}\dfrac{4i}{d}\\[/tex]

[tex]\implies \underline{\underline{\green{ B_{net}=\dfrac{\mu_0i}{ \pi d}}}}\\[/tex]

The direction is down the page .

and we are done!

Which one of the following types of electromagnetic radiation is produced by the sudden deceleration of high speed electrons?
a.x-rays
b.microwaves
c.infrared radiation
d.visible light
e.gamma rays

Answers

The correct answer is a. x-rays is produced by the sudden deceleration of high speed electrons.

What is x-rays?

When high-speed electrons are suddenly decelerated or slowed down, they release energy in the form of electromagnetic radiation. This process is known as bremsstrahlung or "braking radiation". The energy of the emitted radiation depends on the initial speed of the electrons and the degree of deceleration.

In the case of bremsstrahlung, the emitted radiation can range from radio waves to gamma rays, but the highest energy radiation produced by bremsstrahlung is x-rays. Therefore, the sudden deceleration of high-speed electrons produces x-rays.

X-rays are ionizing radiation, meaning that they have enough energy to remove electrons from atoms or molecules, which can cause damage to living tissue. Therefore, exposure to X-rays should be limited and controlled to minimize health risks.

To know more about x-rays, visit:

https://brainly.com/question/29790297

#SPJ1

Complete question is: x-rays is produced by the sudden deceleration of high speed electrons.

For small bodies with high thermal conductivity, the features surrounding the medium that favor lumped system analysis
The medium should be a poor conductor of heat
The medium should be motionless

Answers

Small bodies with high thermal conductivity, the medium should be a poor conductor of heat and should be motionless in order to favour lumped system analysis.

For small bodies with high thermal conductivity, the features surrounding the medium that favor lumped system analysis are that the medium should be a poor conductor of heat and the medium should be motionless.

In other words, for small bodies with high thermal conductivity, the thermal energy will stay confined within the boundaries of the medium if it is a poor conductor of heat and the medium is not moving. This allows the energy to be spread evenly throughout the system, which is why lumped system analysis can be used.

Lumped system analysis is a method used to analyse heat transfer and energy flow within a system. It assumes that thermal energy is transferred across a body of homogeneous material and can be used to calculate the temperature of an object at different points in the body.

The effectiveness of this method relies on the heat capacity of the medium and its thermal conductivity, which is why it is most suitable for small bodies with high thermal conductivity.

For large bodies, or bodies with low thermal conductivity, distributed system analysis is typically used instead of lumped system analysis. This method assumes that the body has different thermal properties at different points, and calculates the temperature at those points based on their respective thermal properties.

For similar questions on thermal conductivity

https://brainly.com/question/29419715

#SPJ11

an electron is moving parallel to an electric field (from higher to lower voltage). its potential energy is

Answers

The potential energy of an electron moving parallel to an electric field decreases as it moves from higher voltage to lower voltage. The work done by the electric field on the electron is equal to the decrease in potential energy. The potential energy of the electron is proportional to its charge and the voltage difference between the two points.

When an electron moves parallel to an electric field, its potential energy is conserved. The potential energy of an electron is proportional to its charge and the voltage through which it moves. As the electron moves from higher voltage to a lower voltage, its potential energy decreases. The work done by the electric field on the electron is equal to the decrease in potential energy. When the electron is at rest, it has a certain potential energy due to its position in the electric field. If the electron is allowed to move freely, it will accelerate towards the lower voltage region, gaining kinetic energy. As it moves, the electric field continues to do work on the electron, converting its potential energy into kinetic energy. If the electric field is uniform, the potential energy of the electron will be given by the equation U = -qV, where q is the charge of the electron and V is the voltage difference between the two points. The negative sign indicates that the potential energy decreases as the voltage difference decreases.

To learn more about Potential energy :

https://brainly.com/question/21175118

#SPJ11

clock a remains in place and clock b is carried around the earth ( 40,000 km). by how many seconds will is clock b slower if carried on

Answers

Clock a remains in place and clock b is carried around the earth (40,000 km). According to Einstein's theory of relativity, The clock b is slower by approximately 44.6 seconds.

According to Einstein's theory of relativity, time dilation takes place when an object moves at a velocity close to the speed of light. The closer the velocity is to the speed of light, the more time slows down. This is why time on Earth is slower at high altitudes than it is on the ground.

According to the theory, the same effect happens when objects are moving at a high speed, which is why clocks that are taken on an airplane, for example, appear to be ticking more slowly.

1. The following equation is used to determine the time dilation:

t = t0 / √(1 – v²/c²),

where t is the time elapsed, t0 is the time at rest, v is the velocity, and c is the speed of light. When the earth rotates on its axis, every point on the planet's surface moves at a different velocity, with the highest velocity at the equator, and the velocity decreases as we move towards the poles. The earth's circumference at the equator is roughly 40,000 kilometers (24,901 miles).
As a result, a person standing on the equator would be traveling at a speed of around 1,674 kilometers per hour (1,040 miles per hour) because the earth spins once every 24 hours. We must first determine the velocity of a point on the earth's surface at the equator before we can use the equation to calculate time dilation.

2. We use the formula

v = 2πr / T,

where v is velocity, r is the radius of the earth, and T is the time it takes the earth to complete one rotation. The formula is as follows:

v = 2πr / Tv

= 2 x 3.14 x 6,378 km / 24 hv

= 1,674 km/h

3. Substituting these values into the equation, we get:

t = t0 / √(1 – v²/c²)t = t0 / √(1 – (1,674 m/s)² / (299,792,458 m/s)²)t = t0 / √(1 – 2.8 x 10^-8)t = t0 / 0.9999999714

This means that the clock on the equator will tick slightly slower than it would at rest. The difference in time can be calculated by subtracting the two values:

t – t0 = t0 / 0.9999999714 – t0t – t0 = t0 (1 – 0.9999999714)t – t0 = 0.0000000286 t0

4. We must first calculate the amount of time elapsed on the equator if a clock b is carried 40,000 km around the earth. It is easy to calculate the distance and speed, but we must also consider that the earth is rotating as well. As a result, we must determine the combined speed of the earth's rotation and the motion of clock b relative to the earth's surface.

5. To calculate this combined velocity, we can use the Pythagorean theorem, which states that the square of the hypotenuse of a right triangle is equal to the sum of the squares of the other two sides. If we imagine the velocity of the earth's rotation as the base of the triangle and the velocity of clock b as the height of the triangle, we can use this theorem to calculate the combined velocity as follows:

combined velocity = √(1,674² + vclock²)

where v clock is the velocity of clock b. Since clock b is being transported at the equator, it has the same velocity as the earth's rotation. As a result, we can substitute 1,674 km/h for v clock:

combined velocity = √(1,674² + 1,674²)

combined velocity = √(2 x 1,674²)

combined velocity = 2,367 km/h

6. Substituting the combined velocity into the equation for time dilation, we obtain:

t – t0 = t0 (1 – √(1 – v²/c²))t – t0 = t0 (1 – √(1 – (2,367 km/h)² / (299,792,458 m/s)²))t – t0

= t0 (1 – √(1 – 1.579 x 10^-11))t – t0

= t0 (1 – 0.999999999920215)t – t0

= 0.000000000079785 t0

Converting this value to seconds, we get:

0.000000000079785 t0 = 79.785 ns

Now we can combine the time dilation for the earth's rotation and the motion of clock b to obtain the total time dilation:

t – t0 = 0.0000000286 t0 + 0.000000000079785 t0t – t0 = 0.000000028679785 t0

Substituting the value of t0 (one second) into the equation, we get:

t – 1 = 0.000000028679785 seconds

Therefore, clock b will be approximately 44.6 seconds slower than clock a after being carried 40,000 km around the earth.

For more such questions on Einstein's theory of relativity , Visit:

https://brainly.com/question/3489672

#SPJ11

Problem 23.13 One type of antenna for receiving AM radio signals is a square loop of wire, 0.16 m on a side, that has 20 turns. Part A If the magnetic field from the radio waves changes at a rate of 8.4 × 10-4 T/s and is perpendicular to the loop, what is the magnitude of the induced emf in the loop? Express your answer to two significant figures and include appropriate units. Value Units Submit My Answers Give Up back Continue

Answers

The induced emf by the formula that we have can be obtained as 4.3 * 10^-4 V.

What is the induced emf?

The induced emf (electromotive force) is the voltage that is generated in a conductor when there is a change in the magnetic field that surrounds the conductor. This phenomenon is known as electromagnetic induction and was discovered by Michael Faraday in the 19th century.

The induced emf is created by the interaction between the magnetic field and the moving charges in the conductor. When the magnetic field changes, it creates an electric field that pushes the charges in the conductor, creating a current flow.

Using emf = NAdB/dt

= 20 * (0.16)^2 *  8.4 × 10-4 T/s

4.3 * 10^-4 V

Learn more about emf:https://brainly.com/question/15121836

#SPJ1

Determine the linear velocity of blood in the aorta with a radis of 1.5 cm, if the duration of systole is 0.25 s, the stroke volume is 60 ml.

Answers

Answer:

The linear velocity of blood in the aorta can be calculated using the equation:

v = Q / A

where v is the linear velocity, Q is the volume flow rate, and A is the cross-sectional area of the vessel.

The volume flow rate Q can be calculated using the equation:

Q = SV / t

where SV is the stroke volume and t is the duration of systole.

The cross-sectional area of the aorta can be calculated using the equation:

A = πr^2

where r is the radius of the aorta.

Given that the radius of the aorta is 1.5 cm, the stroke volume is 60 ml, and the duration of systole is 0.25 s, we can calculate the volume flow rate Q:

Q = SV / t = 60 ml / 0.25 s = 240 ml/s

Converting the units of Q to cm^3/s:

Q = 240 ml/s × 1 cm^3/1 ml = 240 cm^3/s

We can then calculate the cross-sectional area of the aorta:

A = πr^2 = π × (1.5 cm)^2 = 7.07 cm^2

Finally, we can calculate the linear velocity of blood in the aorta:

v = Q / A = 240 cm^3/s / 7.07 cm^2 = 33.9 cm/s

Therefore, the linear velocity of blood in the aorta is 33.9 cm/s.

Find the acceleration vector for the charge. Enter the x, y, and z components of the acceleration in meters per second squared separated by commas. A= m/s^2 To practice Problem-Solving Strategy 27.1: Magnetic Forces. A particle with mass 1.81 xio-3 kg and a charge of 1.22 times sign 10^-8 C has, at a given instant, a velocity v = (3.00 times sign 10^4 m/s)j. What are the magnitude and direction of the particle's acceleration produced by a uniform magnetic field B=(1.63 T)i+(0.980 T)j? Draw the velocity v and magnetic field B vectors. Since they have different units, their relative magnitudes aren't relevant. Be certain they have the correct orientations relative to the given coordinate system. The dot in the center of the image represents the particle. Recall that i, j, and k are the unit vectors in the x, y, and z directions, respectively

Answers

The x, y, and z components of the acceleration are -3.17 x 10^2 m/s^2, -3.17 x 10^2 m/s^2, and -3.17 x 10^-1 m/s^2, respectively.

What is Acceleration?

Acceleration is the rate of change of velocity with respect to time. It is a vector quantity, meaning it has both magnitude and direction. When an object undergoes acceleration, its velocity changes either in magnitude, direction, or both. The formula for acceleration is a = (v_f - v_i) / t, where a is acceleration, v_f is final velocity, v_i is initial velocity, and t is the time taken for the change in velocity.

Using the formula for the magnetic force on a moving charged particle, F = q(v x B), we can find the acceleration vector by dividing the force by the mass of the particle, a = F/m.

The velocity vector v = (0, 3.00 x 10^4, 0) m/s has only a y-component, and the magnetic field vector B = (1.63, 0.980, 0) T has only x- and y-components. Therefore, the cross product of v and B only has a z-component:

v x B = (3.00 x 10^4)i x 0.980j - (3.00 x 10^4)j x 1.63i = -4.71 x 10^7 k m/s

The magnetic force on the charge is then given by:

F = q(v x B) = (1.22 x 10^-8 C)(-4.71 x 10^7 k m/s) = -5.74 x 10^-1 N k

Finally, the acceleration vector is:

a = F/m = (-5.74 x 10^-1 N k)/(1.81 x 10^-3 kg) = (-3.17 x 10^2 i - 3.17 x 10^2 j - 3.17 x 10^-1 k) m/s^2

Learn more about Acceleration from given link

https://brainly.com/question/460763

#SPJ1

A uniform disk with a mass of 190 kg and a radius of 1.1 m rotates initially with an angular speed of 950 rev/min. A constant tangential force is applied at a radial distance of 0.5 m. How much work must this force do to stop the wheel? Answer in units of kJ.

Answers

Answer:

Explanation:

We can use the work-energy principle to find the work done by the applied force to stop the disk. The work-energy principle states that the work done by all forces acting on an object is equal to the change in its kinetic energy:

W = ΔK

where W is the work done, and ΔK is the change in kinetic energy.

Initially, the disk is rotating with an angular velocity of 950 rev/min. We need to convert this to radians per second, which gives:

ω_initial = (950 rev/min) × (2π rad/rev) × (1 min/60 s) = 99.23 rad/s

The initial kinetic energy of the disk is:

K_initial = (1/2) I ω_initial^2

where I is the moment of inertia of the disk about its axis of rotation. For a uniform disk, the moment of inertia is:

I = (1/2) m R^2

where m is the mass of the disk, and R is the radius. Substituting the given values, we get:

I = (1/2) (190 kg) (1.1 m)^2 = 115.5 kg m^2

Therefore, the initial kinetic energy of the disk is:

K_initial = (1/2) (115.5 kg m^2) (99.23 rad/s)^2 = 565201 J

To stop the disk, the applied force must act opposite to the direction of motion of the disk, and must cause a negative change in the kinetic energy of the disk. The force is applied at a radial distance of 0.5 m, which gives a torque of:

τ = F r

where F is the magnitude of the force. The torque causes a negative change in the angular velocity of the disk, given by:

Δω = τ / I

The work done by the applied force is:

W = ΔK = - (1/2) I Δω^2

Substituting the given values, we get:

W = - (1/2) (115.5 kg m^2) [(F r) / I]^2

The force F can be eliminated using the equation for torque:

F = τ / r = (Δω) I / r

Substituting this into the equation for work, we get:

W = - (1/2) (115.5 kg m^2) [(Δω) I / r I]^2

= - (1/2) (115.5 kg m^2) (Δω / r)^2

Substituting the values for Δω and r, we get:

W = - (1/2) (115.5 kg m^2) [(F r / I) / r]^2

= - (1/2) (115.5 kg m^2) [(2 Δω / R) / (2/5 m R^2)]^2

= - (1/2) (115.5 kg m^2) (25/4) (2 Δω / R)^2

= - 90609 J

where we have used the expression for the moment of inertia of a uniform disk and the given values for the mass and radius. The negative sign indicates that the work done by the applied force is negative, which means that the force does negative work (i.e., it takes energy away from the system). The work done by the force to stop the disk is therefore 90609 J, which is -90.6 kJ (to two decimal places).

imagine that the blue light and orange light from the source were blocked. what color would how be present in the spectrum of light observed

Answers

Everything but blue & orange would now be present in the spectrum of light observed.

Spectrum refers to a range of different wavelengths of electromagnetic radiation. Electromagnetic radiation is a form of energy that travels through space and includes different types such as radio waves, microwaves, infrared radiation, visible light, ultraviolet radiation, X-rays, and gamma rays. Each type of electromagnetic radiation has a different wavelength and frequency, and together they make up the electromagnetic spectrum.

The concept of spectrum is used in a variety of fields, including physics, astronomy, and telecommunications. The spectrum of electromagnetic radiation is essential for many technologies, such as radios and televisions, cell phones, and medical imaging devices, as they all rely on the transmission and reception of specific wavelengths of electromagnetic radiation.

To learn more about Spectrum visit here:

brainly.com/question/11837978

#SPJ4

Complete Question: -

Imagine that the blue light and orange light from the source were blocked. What color(s) would now be present in the spectrum of light observed?

I need some help with this problem

Answers

Tensile force refers to the stretching forces that operate on a substance and consists of two components: tensile tension and tensile strain. This indicates that the substance being acted upon is under tension, and the forces are attempting to stretch it.

What Does Tensile Force Mean?

Tensile force refers to the stretching forces that operate on a substance and consists of two components: tensile tension and tensile strain. This indicates that the substance being acted upon is under tension, and the forces are attempting to stretch it.

When a tensile force is applied to a substance, a stress equivalent to the applied force forms, contracting the cross-section and elongating the length.

Learn more about Tensile Force

https://brainly.com/question/17077889

#SPJ1

A diesel engine of a 400-Mg train increases the train's speed uniformly from rest to 10 m/s in 100 s along a horizontal track. Determine the average power developed.

Answers

The average power developed by a diesel engine of a 400-Mg train increases the train's speed uniformly from rest to 10 m/s in 100 s along a horizontal track = 200 kW.

How to calculate average power?

The first kinematic equation is v=v0+at , where v is the final velocity, v0 is the initial velocity, a is the constant acceleration, and t is the time

According to given information:

v = 10, v0= 0 , t= 100s, m=400

v=v0+at

10= 0+a(100)  

a= 0.1 m/s²

∑ F =ma  <==>  F= 400(10 ³ )(0.1) = 40(10 ³)N

Pavg = F. Vavg = 40(10 ³)(10/2) = 200 kW

It represents the typical quantity of work completed or energy converted per unit of time. When the context clearly indicates it, the average power is frequently referred to as "power".

The instantaneous power overrides the average power as time interval t gets closer to zero.

For more information on average power kindly visit to

https://brainly.com/question/17008088

#SPJ1

Use the work energy theorem to rank the final kinetic energy of a ball based on the initial kinetic energy Ki, the magnitude of a constant force F on the ball, the displacement of the ball, d and the angle, theta between the displacement of the ball and the net force on the ball. Rank from greatest kinetic energy (1) to least kinetic energy (4).

a) Ki=150J F=10N d=15m theta=90 degrees
b) Ki=300J F=200N d=1.5m theta=180 degrees
c) Ki=200J F=25N d=4m theta=0 degrees
d) Ki=450J F=15N d=30m theta=150 degrees​

Answers

Answer:

Explanation:

The work-energy theorem states that the net work done on an object is equal to its change in kinetic energy. Therefore, we can use this theorem to calculate the final kinetic energy of the ball in each case.

We know that the work done by a constant force is given by the equation W = Fd cos(theta), where F is the magnitude of the force, d is the displacement of the ball, and theta is the angle between the force and displacement vectors.

Using the work-energy theorem, we can write:

W = ΔK = Kf - Ki

where ΔK is the change in kinetic energy, Kf is the final kinetic energy, and Ki is the initial kinetic energy.

We can rearrange this equation to solve for Kf:

Kf = Ki + W = Ki + Fd cos(theta)

a) Kf = 150 J + (10 N)(15 m)cos(90°) = 150 J

b) Kf = 300 J + (200 N)(1.5 m)cos(180°) = 0 J

c) Kf = 200 J + (25 N)(4 m)cos(0°) = 300 J

d) Kf = 450 J + (15 N)(30 m)cos(150°) = 112.5 J

Ranking from greatest to least final kinetic energy:

c) Ki=200J F=25N d=4m theta=0 degrees

a) Ki=150J F=10N d=15m theta=90 degrees

d) Ki=450J F=15N d=30m theta=150 degrees​

b) Ki=300J F=200N d=1.5m theta=180 degrees

Adult brains are not capable of neurogenesiss . True False

Answers

False. Adult brains are capable of neurogenesis, which is the process of generating new neurons (nerve cells) in the brain. Although it was previously believed that neurogenesis only occurred during early development, research has shown that certain regions of the brain, such as the hippocampus, continue to produce new neurons throughout adulthood. However, the rate of neurogenesis in adults is much lower than in developing brains

Answer:

False. Adult brains are capable of neurogenesis, which is the process of generating new neurons (nerve cells) in the brain. Although it was previously believed that neurogenesis only occurred during early development, research has shown that certain regions of the brain, such as the hippocampus, continue to produce new neurons throughout adulthood. However, the rate of neurogenesis in adults is much lower than in developing brains

EX :SOMEONE FATHER TODAY YOUR FATHER DOES,T KNOW ABOUT TECH OR ANY SAMRT APPS BUT HE KNOW BETTER N HIS GENRATON

A 1.5kg block is held in place and compresses a 150N/m spring by 30cm from its relaxed position. The block is then released. What speed will the block have at the instant when the spring is no longer compressed?

Answers

Answer: simple harmonic motion

Simple harmonic motion. At the instant the spring is no longer compressed(equilibrium), all of our spring potential energy(kx^2/2) has been converted to kinetic energy(mv^2/2). All you have to do is find what your spring potential energy is when the spring is compressed using the spring constant(150N/m) and the distance it's compressed(30cm), use that as your kinetic energy, and solve for the velocity since you already know the mass.

A compact car can climb a hill in 10 s. The top of the hill is 30 m higher than the bottom, and the car’s mass is 1,000 kg What is the power output of the car?

Answers

Answer:

the power output of the car is 29.43 kW (rounded to two decimal places).

Explanation:

To find the power output of the car, we need to use the formula:

power = work / time

where work is the change in potential energy of the car as it climbs the hill, which can be calculated using the formula:

work = force x distance

where force is the force required to lift the car against gravity, which is given by:

force = mass x gravity

where mass is the mass of the car, and gravity is the acceleration due to gravity (9.81 m/s^2).

So, the force required to lift the car against gravity is:

force = 1000 kg x 9.81 m/s^2 = 9810 N

The distance the car travels up the hill is 30 m.

Therefore, the work done by the car is:

work = force x distance = 9810 N x 30 m = 294300 J

The time taken by the car to climb the hill is 10 s.

Therefore, the power output of the car is:

power = work / time = 294300 J / 10 s = 29430 W

Use the work energy theorem to rank the final kinetic energy of a ball based on the initial kinetic energy Ki, the magnitude of a constant force F on the ball, the displacement of the ball, d and the angle, theta between the displacement of the ball and the net force on the ball. Rank from greatest kinetic energy (1) to least kinetic energy (4).

a) Ki=150J F=10N d=15m theta=90 degrees
b) Ki=300J F=200N d=1.5m theta=180 degrees
c) Ki=200J F=25N d=4m theta= 0 degrees
d) Ki=450J F=15N d=30m theta=150 degrees​

Answers

Explanation:

hope its help thank you

follow nyo po me

Where will the temperature most likely be the highest?
A. in a forest
B. in an open field
C. in the shade of a tree
D. in the shadow of a building

Answers

Option A because it’s the hottest out of all of them

Answer:

it's b

Explanation:

no shade, direct sunlight

Other Questions
The government wants to reduce electricity consumption by 10%. The price elasticity of demand for electricity is -5. The government must set the price of electricity atA) raise; 2.0%B) raise; 0.5%C) raise; 1.25%D) lower; 0.5%. if you move your watchband up your wrist an inch or so, you will feel it for only a few moments. this best illustrates: responses weber's law weber's law parallel processing parallel processing accommodation accommodation sensory adaptation Researchers have identified a series of speciation events that have created a new group of organisms. Which of the following terms best describes what has occurred?A) MacroevolutionB) BiogeographyC) MicroevolutionD) Geologic time scale Complete the below table to calculate the price of a $1.5 million bond issue under each of the following independent assumptions (FV of $1, PV of $1, FVA of $1, PVA of $1, FVAD of $1 and PVAD of $1) (Use appropriate factor(s) from the tables provided.Enter your answers in whole dollars.):1.Maturity 15 years, interest paid annually, stated rate 10%, market rate 12%Table values are based on:n=i=Cash FlowAmountPresent ValueInterestPrincipalPrice of bonds2.Maturity 6 years, interest paid semiannually, stated rate 14%, market rate 16%Table values are based on:n=i=Cash FlowAmountPresent ValueInterestPrincipalPrice of Bonds3.Maturity 5 years, interest paid semiannually, stated rate 16%, market rate 14%Table values are based on:n=i=Cash FlowAmountPresent ValueInterestPrincipalPrice of bonds4.Maturity 20 years, interest paid semiannually, stated rate 14%, market rate 16%Table values are based on:n=i=Cash FlowAmountPresent ValueInterestPrincipalPrice of Bonds5.Maturity 20 years, interest paid semiannually, stated rate 14%, market rate 14%Table values are based on:n=i=Cash FlowAmountPresent ValueInterestPrincipalPrice of bonds two people standing at different locations are looking at a tall building. person a angle of elevation to the building is 35 degrees. person b angle of elevation is 77 degrees. the building is 8 miles away from person b. how far away is person a from the building? TRUE/FALSE. the usa patriot act (2001) gives individuals more protection from governmental intrusion. Write each equation in slope-intercept form. Identify the slope and y-intercept.x - 3y = 12*Work must be shown.* What is result of following operation(4623. 56)10+ (110011. 11)2whare (110011. 11(2 mean that 110011. 11as a number express in base 2 many tests to distinguish aldehydes and ketones involve the addition of an oxidant. only choose... can be easily oxidized because there is choose... next to the carbonyl and oxidation does not require choose... Mrs Smith walks a half a mile a day after work. She works five days a week. How many yards will she have walked for the week by Friday morning? In which of the following groups of trade blocs does most of today's world trade take place?A) Middle East, China, and IndiaB) North America, Africa, and CanadaC) Western Europe, Asia, and the AmericasD) Saudi Arabia, Western Europe, and the Gulf (01.05 MC)Danielle is writing a thesis statement about the impact of indirect characterization on the development of the plot. Read her thesis statement and answer the question that follows.In "The Ascent" by Ron Rash, there are many things that the author does with the people in the story to make the plot go.What should Danielle most focus on when revising her thesis statement for formal language? Which of the following quotations best represents the thesis statement of the passage?"But reflecting persons perceived that when society is itself the tyrantsociety collectively over the separate individuals who compose itits means of tyrannising are not restricted to the acts which it may do by the hands of its political functionaries." (paragraph 1, sentence 2)"Protection, therefore, against the tyranny of the magistrate is not enough; there needs protection also against the tyranny of the prevailing opinion and feeling . .." (paragraph 1, sentence 4)"But though this proposition is not likely to be contested in general terms, the practical question, where to place the limithow to make the fitting adjustment between individual independence and social controlis a subject on which nearly everything remains to be done." (paragraph 2, sentence 1)"All that makes existence valuable to anyone depends on the enforcement of restraints upon the actions of other people." (paragraph 2, sentence 2)"No two ages, and scarcely any two countries, have decided it alike; and the decision of one age or country is a wonder to another." (paragraph 2, sentence 5) Your local barista recommends a new Thai restaurant that just opened down the street. This exchange demonstrates the usefulness of __________ in relationships. facework empathy weak ties relational listening the nutty professor sells cashews for $7.70 per pound and brazil nuts for $4.80 per pound. how much of each type should be used to make a 27 pound mixture that sells for $6.41 per pound? Flextime is a system that allows employees some freedom to choose which hours to work as long as they:- work schedule that gives employees some freedom- work the required number of hours- complete their assigned tasks big fish presentations, the company profiled in the opening feature of chapter 13, prepares presentations that help clients present themselves in new and innovative ways. in regard to growth, big fish presentations has opted to HELP PLEASE LATE HOMEWORK DUE TODAY a. You are Ankit Sharma staff reporter of HindustanTimes. You were asked to cover a trainaccident. Mentioning all relevant details writereport. (100-120 words) 1. rayah reminds herself that when receiving negative feedback, she has a propensity to blame others for her mistakes. (click to select) 2. rainer decides to hide his anger after receiving negative feedback from his superior and he stops listening to the feedback. (click to select) 3. edie focuses on what she might be able to learn from her boss during her performance review. (click to select) 4. francis schedules a meeting with her boss to ask if he would list one thing that he thinks she could do a better job with on her current project. (click to select)